Đến nội dung

moonkey01 nội dung

Có 48 mục bởi moonkey01 (Tìm giới hạn từ 19-04-2020)



Sắp theo                Sắp xếp  

#680292 Topic ôn thi hình học vào cấp 3 chuyên

Đã gửi bởi moonkey01 on 11-05-2017 - 14:15 trong Hình học

Mình xin được có một vài ý kiến.

 

Thiết nghĩ việc sử dụng các bài toán Olympic quá khó cho việc ôn luyện vào THPT chuyên không phải là một việc làm hay. Như bài toán của thầy Nguyễn Minh Hà được lấy từ mục "Thách đấu" trên tạp chí TTT2 tuy lời giải sử dụng kiến thức THCS nhưng không thích hợp trong điều kiện đề thi của THCS. Mình cho rằng với kỳ thi vào lớp chuyên, một bài toán hình học có ý nghĩa là khi không quá dễ, phát biểu đẹp nhưng cũng đồng thời kiểm tra được khả năng đoán nhận tính chất và nhận biết vấn đề của học sinh trong thời gian ngắn. Còn về những tính chất khó đoán nhận hơn, đó lại là câu chuyện sau khi vượt qua kỳ thi đó. Và thật ra đa số các bài toán Olympic đều có lời giải chỉ dùng kiến thức THCS.

 

Bài toán 91. Cho tam giác $ABC$ nhọn nội tiếp $(O)$. Tiếp tuyến tại $B,C$ của $(O)$ cắt nhau ở $K$. Gọi $I$ đối xứng $O$ qua $BC$. $L$ là trung điểm $OK$. Chứng minh rằng $\angle IAB=\angle LAC$.




#676741 Kì thi Olympic tháng 4 TP.HCM lần 3 2016-2017 lớp 10

Đã gửi bởi moonkey01 on 09-04-2017 - 14:45 trong Thi HSG cấp Tỉnh, Thành phố. Olympic 30-4. Đề thi và kiểm tra đội tuyển các cấp.

Bài toán Hình học không mới và đã từng xuất hiện ở đây:

 

http://analgeomatica...-2-thang-3.html

 

Trong thời gian thi hạn chế mà độ khó bài hình học thế này thì thật sự không đẹp lắm. Không khác gì việc lấy bài kiểm tra đội tuyển IMO năm 2015 ra để làm đề thi chính thức năm vừa rồi.




#674692 ĐỀ THI HỌC SINH GIỎI LỚP 10 chuyên sư phạm 2016-2017

Đã gửi bởi moonkey01 on 18-03-2017 - 23:18 trong Thi HSG cấp Tỉnh, Thành phố. Olympic 30-4. Đề thi và kiểm tra đội tuyển các cấp.

Một lời giải khác cho bài toán $2$ ngày II.

 

Để thuận tiện, ta quy ước ký hiệu $\sum f(a,b,c)=f(a,b,c)+f(b,c,a)+f(c,a,b)$ thì vế trái có thể viết lại thành:

 

$\sum \left(\frac{a}{b+c-a}+\frac{1}{2}\right)=\sum \frac{a+b+c}{2(b+c-a)}=\frac{a+b+c}{2}\cdot \sum \frac{1}{b+c-a}$

 

Do đó ta chỉ cần chứng minh rằng $\sum \frac{1}{b+c-a}\geq \frac{(a+b+c)^2}{3abc}$. Thật vậy, theo bất đẳng thức Cauchy-Schwarz:

 

$\sum \frac{1}{b+c-a}\geq \frac{(a+b+c)^2}{\sum a^2(b+c)-\sum a^3}\geq \frac{(a+b+c)^2}{3abc}$

 

Trong đó đánh giá cuối cùng đúng theo bất đẳng thức Schur bậc $3$. Bài toán được chứng minh. $\square$




#674689 ĐỀ THI HỌC SINH GIỎI LỚP 10 chuyên sư phạm 2016-2017

Đã gửi bởi moonkey01 on 18-03-2017 - 23:00 trong Thi HSG cấp Tỉnh, Thành phố. Olympic 30-4. Đề thi và kiểm tra đội tuyển các cấp.

Đây là nguyên văn đề thi, MS tạm bỏ phần hướng dẫn giải để các bạn làm.

 

Lời giải của em cho bài toán 4 ngày II:

 

Ta gọi $3$ số được viết lên bảng qua mỗi thao tác thực hiện trên $1$ cặp số là một "bộ ba" được tạo thành từ cặp số đó. Với $1$ cặp số nguyên dương, có các khả năng sau cho một bộ ba:

- Nếu chọn ra $2$ số lẻ (tạm gọi là "cặp lẻ") thì bộ ba có đúng $1$ số lẻ.

- Nếu chọn ra $2$ số chẵn (tạm gọi là "cặp chẵn") thì bộ ba không có số lẻ nào.

- Nếu chọn ra $1$ số chẵn, $1$ số lẻ (tạm gọi là "cặp xấu") thì bộ ba có đúng $2$ số lẻ.

 

Gọi ban đầu ta có $x$ số lẻ và $7-x$ số chẵn thì:

- Số lượng số lẻ tạo ra từ $7-x$ số chẵn, nghĩa là từ $C_{7-x}^{2}$ cặp chẵn là $0$ số.

- Số lượng số lẻ tạo ra từ $x$ số lẻ, nghĩa là từ $C_{x}^{2}$ cặp lẻ là tối đa $C_{x}^{2}=\frac{x(x-1)}{2}$ số.

- Số lượng số lẻ tạo ra từ $x$ số lẻ và $7-x$ số chẵn, nghĩa là từ $x(7-x)$ cặp xấu là tối đa $2x(7-x)$ số.

 

Từ đó $\left|G\right|\leq \frac{x(x-1)}{2}+2x(7-x)=\frac{3x(9-x)}{2}$. Do $0\leq x\leq 7$ và $x\in \mathbb{N}$ nên bằng biến đổi đơn giản, ta có $\frac{3x(9-x)}{2}\leq 30$ hay $G\leq 30$.

 

Đẳng thức xảy ra chẳng hạn khi $x=4$ và ta chọn $7$ số ban đầu là:

$a_1=10^1+1,a_3=10^3+1,a^5=10^5+1,a_7=10^7+1$ và $a_2=10^2,a_4=10^4,a_6=10^6$.

 

Vậy giá trị lớn nhất của $\left|G\right|$ là $30$. $\square$




#673021 VMF's Marathon Hình học Olympic

Đã gửi bởi moonkey01 on 28-02-2017 - 20:16 trong Hình học

Anh Hiếu nhờ em đề xuất tiếp bài toán sau đây:

 

Bài toán 179: Cho tam giác $ABC$ có đường cao $BE,CF$. $M$ là hình chiếu của $A$ trên $EF$. $N,P$ là trung điểm $BE,CF$. Chứng minh rằng hai tam giác $ABC,MNP$ đồng dạng.




#672741 VMF's Marathon Hình học Olympic

Đã gửi bởi moonkey01 on 25-02-2017 - 19:29 trong Hình học

Bài toán 174. Cho $\Delta ABC$ có 1 điểm $D$ bất kì thuộc đường thẳng $BC$ sao cho $D$ và $B$ nằm khác phía so với $C$. Gọi $I$ và $I_1$ l ần lượt là tâm nội tiếp $\Delta ABC$ và $\Delta ACD$. Chứng minh rằng trục đẳng phương của $(I)$ và $(I')$ đi qua 1 điểm cố định khi $D$ thay đổi.

 

Đây là bài toán IMO Shortlist 2004, có thể xem tại đây: https://artofproblem...c6h41033p257883

 

Bài toán sau trích từ chuỗi bài giảng của thầy Hùng tại trường Đông miền Nam 2015:

 

Bài toán 175. Cho tam giác $ABC$ nội tiếp $(O)$ có trực tâm $H$. $AD,AM$ lần lượt là đối trung và trung tuyến. Gọi $P,Q$ đối xứng $H$ qua $AD,AM$ và $R$ đối xứng $H$ qua $D$. Chứng minh rằng $(PQR)$ tiếp xúc $(O)$.




#671688 VMF's Marathon Hình học Olympic

Đã gửi bởi moonkey01 on 15-02-2017 - 12:28 trong Hình học

Bài toán 170: Cho $\triangle ABC$. $M\in (BC)$ sao cho $M$ nằm trog $\triangle ABC$. Gọi $D,E,F$ lần lượt là hình chiếu của $M$ trên $BC,CA,AB$. $ME$ cắt $AB$ tại $P$, $MF$ cắt $AC$ tại $Q$, $PQ$ cắt $BC$ tại $K$. $H$ đối xứng với $M$ qua trung điểm của $PQ$. CMR: $HK\perp AD$

 

Lời giải sau tham khảo từ Nguyễn Lê Phước trên Facebook.

 

170.JPG

 

Lời giải: Ta có hai tam giác $PBM$ và $QMC$ đồng dạng g.g do $\angle PBM=90^{\circ}-\angle BMF=\angle QMC$ và $\angle BPM=\angle MQC$, từ đó $\frac{PB}{QM}=\frac{PM}{QC}$. Dễ thấy rằng $MPHQ$ là hình bình hành nên $MQ=HP$ và $MP=HQ$, từ đó $\frac{PB}{PH}=\frac{QH}{QC}$ hay tam giác $PBH$ đồng dạng tam giác $QHC$ (c.g.c). Gọi $R$ là hình chiếu của $H$ trên $BC$ thì $\angle PRQ=\angle PRH+\angle QRH=\angle PBH+\angle QCH=90^{\circ}$, lại theo một bài toán quen thuộc là $ND=NR$ nên các điểm $D,E,F,P,Q,R$ thuộc một đường tròn. Gọi $HK$ cắt $(APQ)$ tại $S$ thì $KD.KR=KP.KQ=KH.KS$ dẫn đến $DSHR$ nội tiếp. Do đó $\angle DSH=90^{\circ}=\angle ASH$ nên $A,S,D$ thẳng hàng. Vậy $AD\perp HK$ $\blacksquare$

 

Một kết quả thú vị cho bài toán: $M.H$ là hai điểm đẳng giác trong tam giác $ABC$.




#671275 VMF's Marathon Hình học Olympic

Đã gửi bởi moonkey01 on 12-02-2017 - 14:42 trong Hình học

Theo đề nghị của anh Phương, em đề xuất tiếp bài toán sau của thầy Hùng.

 

Bài toán 169. Cho tam giác $ABC$ có $D$ là điểm bất kỳ trên đường cao từ $A$. Đường tròn $(K)$ đường kính $AD$ cắt $CA,AB$ tại $E,F$. Tiếp tuyến tại $E,F$ của $(K)$ cắt $BC$ tại $M,N$. Gọi $EB$ cắt $FC$ tại $P$, $EN$ cắt $FM$ tại $Q$. Chứng minh rằng $PQ$ luôn đi qua điểm cố định khi $D$ thay đổi.




#671119 VMF's Marathon Hình học Olympic

Đã gửi bởi moonkey01 on 11-02-2017 - 18:19 trong Hình học

Theo đề nghị của anh Khánh, em đề xuất bài toán tiếp theo, có lẽ là của anh Phạm Hy Hiếu, HCB IMO năm 2009.

 

Bài toán 168. Cho tam giác $ABC$ nội tiếp $(O)$ có $BC>CA>AB$ và $I$ là tâm nội tiếp. $AI$ cắt lại $(O)$ tại $K$. $M$ là trung điểm $BC$. Gọi $N$ đối xứng với $I$ qua $M$. $KN$ cắt lại $(O)$ tại $L$. Chứng minh rằng $LB=LC+LA$.




#670873 VMF's Marathon Hình học Olympic

Đã gửi bởi moonkey01 on 09-02-2017 - 19:40 trong Hình học

Được sự đồng ý của thầy Hùng, em đề nghị bài toán tiếp theo, cũng là 1 bài của thầy.

 

Bài toán 166. Cho tam giác $ABC$ nhọn nội tiếp đường tròn $(O)$ bán kính $R$ có $I$ là tâm nội tiếp, $AD$ là đường cao. $K$ là điểm trên tia $AD$ sao cho $AK=2R$. $M$ là hình chiếu của $B$ trên $IK$. Gọi $AD$ cắt lại $(O)$ tại $N$. Giả sử rằng $IK\parallel AB$. Chứng minh rằng $MN\parallel ID$.




#670085 Bài toán chia đôi cạnh trên đường tròn nội tiếp

Đã gửi bởi moonkey01 on 27-01-2017 - 00:38 trong Hình học

Cho tam giác $ABC$ có đường tròn nội tiếp $(I)$ tiếp xúc $BC,CA,AB$ tại $D,E,F$. $AI$ cắt $DE,DF$ tại $M,N$. $P,R$ là trực tâm các tam giác $IAB,IAC$; $BI$ cắt $FM$ tại $Q$ và $CI$ cắt $EN$ tại $S$. Gọi $PQ$ cắt $RS$ tại $T$. Chứng minh rằng $ID$ chia đôi đoạn $AT$.

Bài này được em tình cờ tìm ra cũng lâu trong lúc giải bài toán 102 ở VMF Marathon, đã thử đo đạc và đúng :D



#670006 VMF's Marathon Hình học Olympic

Đã gửi bởi moonkey01 on 26-01-2017 - 14:11 trong Hình học

Bài toán 145 (AoPS). Cho tam giác $ABC$ với $\angle BAC<45^{\circ}$. $D$ ở trong tam giác $ABC$ sao cho $BD=CD$ và $\angle BDC=4\angle BAC$. $E$ là đối xứng của $C$ qua $AB$. $F$ là đối xứng của $B$ qua $AC$. Chứng minh rằng $AD\perp EF$.

 

Lời giải của em như sau:

 

Lời giải bài toán 145: Gọi $O$ là tâm ngoại tiếp tam giác $ABC$ thì $D$ là tâm ngoại tiếp tam giác $OBC$. Đặt $BC=a,CA=b,AB=c$ và bán kính của $(ABC)$, $(OBC)$ lần lượt là $R,r$, ta sẽ chứng minh rằng $DE^2-DF^2=AE^2-AF^2=b^2-c^2$. Thật vậy, dễ dàng tính được $\angle DBE=\angle A+90^{\circ}+\angle B-\angle C$ và $\angle DBE=\angle A+90^{\circ}+\angle C-\angle B$.

 

Lại có các đẳng thức $DE^2=a^2+r^2-2ar\cdot cos\angle DBE, \\DF^2=a^2+r^2-2ar\cdot cos\angle DCF$ và $r=\frac{a}{2sin\angle 2A}=\frac{a}{4sin\angle A\cdot cos\angle A}$ nên ta có:

 

$DE^2-DF^2=2ar(cos\angle DCF-cos\angle DBE)\\= \frac{-a^2\cdot sin(\angle A+90^{\circ})\cdot sin(\angle C-\angle B)}{sin\angle A\cdot cos\angle A}\\=4R^2\cdot sin\angle A\cdot sin(\angle C-\angle B)$

 

Mặt khác $b^2-c^2=2R^2(cos\angle 2C-cos\angle 2B)=4R^2sin(\angle C+\angle B)sin(\angle C-\angle B)$, dẫn đến $DE^2-DF^2=b^2-c^2=AE^2-AF^2$ hay $AD\perp EF$ $\blacksquare$

Hình gửi kèm

  • Ảnh chụp Màn hình 2017-01-26 lúc 14.14.40.png



#668340 Đề thi chọn đội dự tuyển lớp 10 PTNK - ĐHQGTPHCM

Đã gửi bởi moonkey01 on 14-01-2017 - 23:37 trong Thi HSG cấp Tỉnh, Thành phố. Olympic 30-4. Đề thi và kiểm tra đội tuyển các cấp.

Cho mình hỏi đây là đề lớp 10 va 11 à

 

Đây là đề dành cho khối 10 để chọn đội dự tuyển trường PTNK; lời giải trên được đưa ra sau khi kết thúc buổi thi và các anh khoá trước cùng ngồi giải đề với một số bạn khối 10. Cảm ơn sự quan tâm của bạn.




#668337 Đề thi chọn đội dự tuyển lớp 10 PTNK - ĐHQGTPHCM

Đã gửi bởi moonkey01 on 14-01-2017 - 23:21 trong Thi HSG cấp Tỉnh, Thành phố. Olympic 30-4. Đề thi và kiểm tra đội tuyển các cấp.

hướng làm bài 2: theo Phương Trình hàm Cauchy và điều 1 có $f(x)=x^{c} (c \in \mathbb{N})$ dựa theo điều 2 ta suy ra $c$ lẻ . dựa theo điều 3 thì $c=3$ kết luận $f(x)=x^{3}$

 

Vậy bạn có thể phát biểu lại rõ ràng giúp mình rằng như thế nào là phương trình hàm Cauchy và các điều kiện của phương trình hàm Cauchy là gì hay không ?

 

Lời giải sau được đề nghị bởi Lâm Hữu Phúc, học sinh lớp 11 Toán trường PTNK - ĐHQG TP.HCM.

 

Nhận xét rằng vai trò của số $2017$ trong bài toán là không cần thiết cho nên ta sẽ giải bài toán khi thay $2017$ bởi số nguyên dương $p$ bất kỳ. Từ điều kiện đầu tiên, ta có được $f(p^k)=p^{3k}$ với $k$ là số nguyên dương bất kỳ. Tại điều kiện thứ hai, thay $n$ bởi $m$, ta có $f(m)$ là bội của $m$ với mỗi $m$ nguyên dương nên ta đặt $f(m)=m.g(m)$ ($g:\mathbb{N^{*}}\rightarrow \mathbb{N^{*}}$). Khi đó ta có các điều kiện sau:

 

i) $g(mn)=g(m).g(n) \forall m,n \in\mathbb{N^{*}}$

ii) $m.g(m)+n.g(n)$ là bội của $m+n$.

iii) $g(p^{n})=p^{2n} \forall n\in \mathbb{N^{*}}$.

 

Đặt $h(m)=g(m)-m^2$ ($h:\mathbb{N^{*}}\rightarrow \mathbb{Z}$) và thay $n$ bởi $p^n$ tại ii), ta có $m.h(m)$ là bội của $m+p^n$. Chọn $n$ đủ lớn thì $h(m)=0$ với mỗi $m$ hay $f(m)=m^3$ với mỗi $m$ nguyên dương. Thử lại thoả mãn.

 

Kết luận: $f(m)=m^3$ là nghiệm hàm duy nhất. $\blacksquare$




#667956 VMF's Marathon Hình học Olympic

Đã gửi bởi moonkey01 on 11-01-2017 - 00:11 trong Hình học

 

$\boxed { Bài \ toán \ 110} $ (sách) Cho $\triangle ABC$ nội tiếp $(O)$ , $K,L$ là tâm bàng tiếp góc $\angle B, \angle C$. $(K)$ tiếp xúc $BA,BC$ tại $B_1,B_2$, tương tự ta có $C_1,C_2$. $B_1B_2$ cắt $C_1C_2$ tại $N$. Chứng minh rằng $AN \perp BC$

 

 

Lời giải của em như sau. Em tính hết ra.

 

Không mất tính tổng quát, giả sử vị trí các điểm như hình vẽ. Đặt $BC=a, CA=b, AB=c$. Gọi đường cao $AH$ của tam giác $ABC$ cắt $CL,BK,B_1B_2,C_1C_2$ tại $X,Y,T,T'$. Ta có $\angle HT'C_2=\frac{\angle ACB}{2}=\angle ACL$ nên $CXC_1T'$ nội tiếp hay $AX.AT'=AC.AC_1=b(p-b)$. Tương tự thì $AY.AT=AB.AB'=c(p-c)$ nên $\frac{AX}{AY}.\frac{AT'}{AT}=\frac{b(p-b)}{c(p-c)}$. Ta tính được $BH=AB.cos\angle ABC=\frac{c(c^2+a^2-b^2)}{2ca}=\frac{c^2+a^2-b^2}{2a}$, mặt khác theo tính chất đường phân giác thì $\frac{AY}{AB}=\frac{YH}{BH}=\frac{AH}{AB+BH}$ nên $AY=\frac{AB.AH}{AB+BH}=\frac{c.AH}{c+\frac{c^2+a^2-b^2}{2a}}=\frac{2ac.AH}{(c+a-b)(c+a+b)}=\frac{ac.AH}{2p(p-b)}$, tương tự thì $AX=\frac{ab.AH}{2p(p-c)}$. Do đó $\frac{AX}{AY}=\frac{b(p-b)}{c(p-c)}=\frac{AX}{AY}.\frac{AT'}{AT}$, dẫn đến $AT=AT'$ hay $B_1B_2,C_1C_2$ cắt nhau trên $AH$. Vậy $AN\perp BC$ $\blacksquare$

 

Và em muốn nhờ thầy Hùng đề nghị bài toán mới :)

Hình gửi kèm

  • Ảnh chụp Màn hình 2017-01-10 lúc 23.52.43.png



#667876 VMF's Marathon Hình học Olympic

Đã gửi bởi moonkey01 on 10-01-2017 - 15:09 trong Hình học

Bài toán 108 (Mở rộng ý a) bài toán 7 VMO 2017). Cho tam giác $ABC$ nội tiếp đường tròn $(O)$. Một đường tròn $(K)$ đi qua $B,C$. Trung trực $BC$ cắt $(K)$ tại $M,N$. $P$ là điểm thuộc $(K)$. $PM$ cắt $CA,AB$ tại $E,F$. $BE$ cắt $CF$ tại $L$. Chứng minh rằng $AL,PN$ và $BC$ đồng quy.

 

attachicon.gifFigure4244.png

 

Không rõ lời giải của em có nhầm lẫn gì không, nhưng em thấy lời giải khá ngắn.

 

Gọi $NP$ cắt $BC$ tại $T$, $EF$ cắt $BC$ tại $S$ thì $PN$ là phân giác $\angle BPC$ nên $(ST,BC)=-1$, từ đó dễ dàng có ngay $AL$ đi qua $T$.

 

Bài toán tiếp theo là của thầy Hùng, là bài toán 25 trong tập bài giảng của thầy tại đây: http://analgeomatica...t-chuyen-i.html. Nói ngoài lề một ít, đây là tập bài giảng làm em rất ấn tượng, một phần vì các bài sau rất khó, và mỗi lần sau một khoảng thời gian giải lại thì lại tìm thấy những điều thú vị.

 

Bài toán 109 (Sáng tác từ Serbia 2013 và Iran 2015): Cho tam giác $ABC$ nội tiếp $(O)$, $X$ là điểm trên cung nhỏ $BC$ sao cho nếu $E,F$ là hình chiếu của $X$ trên $IB,IC$ thì trung điểm $EF$ nằm trên trung trực $BC$. Gọi $J$ là tâm bàng tiếp góc $\angle BAC$ của tam giác $ABC$. Chứng minh rằng $XJ$ đi qua trung điểm cung lớn $BC$ của $(O)$.




#667657 VMF's Marathon Hình học Olympic

Đã gửi bởi moonkey01 on 08-01-2017 - 20:33 trong Hình học

Bài toán 105.(Mở rộng ý a) VMO 2017 bài toán 3)Cho tam giác $ABC$ nội tiếp đường tròn $(O)$. Một đường tròn $(K)$ đi qua $B,C$ cắt $CA,AB$ tại $E,F$ khác $B,C$. $BE$ cắt $CF$ tại $H$. $AH$ cắt $(O)$ tại $D$ khác $A$. Tiếp tuyến tại $E,F$ của $(K)$ lần lượt cắt $DB,DC$ tại $M,N$. Chứng minh rằng $MN\perp OH$.

 

Bổ đề: Cho tam giác $ABC$ nội tiếp $(O)$. Đường tròn $(K)$ đi qua $B,C$ cắt $CA,AB$ tại $E,F$. $BE$ cắt $CF$ tại $H$. Gọi $AT$ là đường kính của $(O)$. Khi đó $T,K,H$ thẳng hàng.

 

Lời giải bài toán 105: Gọi $L$ là giao điểm của $ME,AH$ thì ta có $\angle LEB=\angle ACB=\angle ADB$ nên $BDEL$ nội tiếp, dẫn đến $HL.HD=HE.HB=HF.HC$ hay $CDFL$ nội tiếp. Từ đó $\angle LFC=\angle ADC=\angle ADC$ nên $FL$ là tiếp tuyến của $(K)$ và $N,F,L$ thẳng hàng. Mặt khác $ML.ME=MB.MD$ và $NL.NF=NC.ND$ nên $MN$ chính là trục đẳng phương của $(LEF)$ và $(K)$, nên ta chỉ cần chứng minh tâm của $(LEF)$ nằm trên $OH$ hay $OH$ chia đôi $LK$. Gọi $AT$ là đường kính của $(O)$, do $BCEF$ nội tiếp nên dễ dàng chứng minh được $EF\perp AT$, mà $EF||LK$ (tính chất tiếp tuyến) nên $LK||EF$. Theo bổ đề, $T,K,H$ thẳng hàng mà $OH$ chia đôi $AT$ nên $OH$ chia đôi $LK$. Vậy $MN\perp OH$.

 

Một hệ quả của bài toán là $MN,EF,BC$ đồng quy, Thật vậy, nếu gọi $EF$ cắt $BC$ tại $S$ thì $SE.SF=SB.SC$ nên $S$ nằm trên trục đẳng phương của $(LEF)$ và $(K)$ hay $MN,EF,BC$ đồng quy.

 

Hình vẽ bài toán 105 ở phần dưới.

 

Bài toán 106 (Sưu tầm): Cho tam giác $ABC$ nhọn có $AD,BE,CF$ là đường cao và trực tâm $H$. $I$ là tâm nội tiếp tam giác $AEF$. Gọi $M,N$ là điểm trên $(IAB),(IAC)$ sao cho $BM,CN$ là phân giác $\angle ABC, \angle ACB$. Chứng minh rằng trung tuyển qua $M,N$ của tam giác $MHB,NHC$ cắt nhau trên $(DEF)$.

Hình gửi kèm

  • Ảnh chụp Màn hình 2017-01-08 lúc 18.41.17.png



#667102 VMF's Marathon Hình học Olympic

Đã gửi bởi moonkey01 on 05-01-2017 - 17:10 trong Hình học

Em giải thích rõ hơn phần tính toán để có được $IN||AF$.

 

Đặt $BC=a,CA=b,AB=c(b\neq c)$ thì dễ dàng có $\frac{IE}{IA}=\frac{a}{b+c}$. Mặt khác cũng tính được $BE=\frac{ca}{c+b}$ nên $NE=BN-BE=\frac{a}{2}-\frac{ca}{c+b}=\frac{a(b-c)}{2(b+c)}$. Hơn nữa $NF=NC-CF=\frac{a}{2}-\frac{c+a-b}{2}=\frac{b-c}{2}$ nên $\frac{NE}{NF}=\frac{a(b-c)}{2(b+c)}:\frac{b-c}{2}=\frac{a}{b+c}=\frac{IE}{IA}$. Theo định lý Thales đảo, $IN||AF$.

 

Bài toán 103 (Mathematical Refections 6/2014). Cho tam giác $ABC$ nội tiếp $(\Gamma)$ có $M$ là điểm chính giữa cung $BC$ không chứa $A$. Gọi $l_{b},l_{c}$ là đường thẳng qua $B,C$ song song $AM$ và cắt lại $(\Gamma)$ tại $P,Q$. $PQ$ cắt $AB,AC$ tại $X,Y$. $AM$ cắt lại $(AXY)$ tại $N$. Chứng minh rằng trung trực $BC,XY,MN$ đồng quy.




#667097 VMF's Marathon Hình học Olympic

Đã gửi bởi moonkey01 on 05-01-2017 - 15:51 trong Hình học

 

Bài toán 102 (Tập huấn đội IMO 2016)Cho tam giác $ABC$ nội tiếp đường tròn $(O)$ cố định. $B$, $C$ cố định, $A$ di chuyển trên $(O)$. $I$ là tâm đường tròn nội tiếp tam giác $ABC$. $K$, $L$ theo thứ tự là trực tâm các tam giác $IAB$, $IAC$. $P$ đối xứng với $O$ qua trung điểm $KL$. Chứng minh rằng $AP$ đi qua một điểm cố định khi $A$ thay đổi.

 

Lời giải của em như sau.

 

Trước hết, ta phát biểu hai bổ đề sau:

 

Bổ đề 1: Hai tam giác đồng dạng có hai cặp cạnh tương ứng vuông góc thì cặp cạnh còn lại cũng vuông góc.

 

Bổ đề 2: $KL,AF$ vuông góc trên $(I)$ với $F$ là tiếp điểm đường tròn bàng tiếp góc $\angle BAC$ của tam giác $ABC$.

 

Hai bổ đề trên đều quen thuộc nên không trình bày lại phép chứng minh tại đây, riêng bổ đề 2 chính là hệ quả bài toán 3 trong kỳ thi Olympic Sharygin vòng 1 năm 2017. Quay trở lại bài toán ban đầu. Do trường hợp $AB=AC$ là hiển nhiên nên không mất tính tổng quát, ta xét trường hợp $AB<AC$ và vị trí các điểm như hình vẽ.

 

Do $K,L$ là trực tâm các tam giác $IAB,IAC$ và $I$ là tâm nội tiếp tam giác $ABC$ nên ta có:

$\angle KAL=\angle KAB+\angle LAC-\angle BAC=90^{o}-\frac{\angle ABC}{2}+90^{o}-\frac{\angle BCA}{2}-\angle BAC=90^{0}-\frac{\angle BAC}{2}$

 

Gọi $Q$ là điểm sao cho $AQ\perp BC$ và $QP||AL$ thì ta có:

$\angle KAQ=\angle BAK-\angle BAQ=90^{o}-\frac{\angle ABC}{2}-(90^{o}-\angle ABC)=\frac{\angle ABC}{2}=\angle IBC$.

 

Mặt khác do $\angle AKQ=180^{o}-\angle KAL=180^{o}-(90^{o}-\frac{\angle BAC}{2})=90^{o}+\frac{\angle BAC}{2}=\angle BIC$ nên tam giác $AKQ$ đồng dạng tam giác $BIC$ (g.g). Gọi $M,N$ là trung điểm $AQ,BC$ thì tam giác $AKM$ đồng dạng tam giác $BIN$ (c.g.c). Từ bổ đề dễ dàng có $KM\perp IN$, lại có $KL\perp AF$ nên ta chỉ cần chứng minh rằng $IN||AF$. Nhưng điều này đúng do dễ dàng tính toán theo 3 cạnh tam giác $ABC$ bởi tính chất phân giác để có tỉ lệ $\frac{IE}{IA}=\frac{BC}{CA+AB}=\frac{NE}{NF}$. Từ đó dẫn đến $KM\equiv KL\perp IN$, lại có $\Delta MAL=\Delta MQK$ (g.c.g) nên $ML=MK$ hay $M$ là trung điểm $KL$ hay $M$ cũng là trung điểm $OP$. Lại có $\frac{AQ}{BC}=\frac{AK}{BI}=cot\frac{\angle BAC}{2}$ không đổi nên $AQ$ không đổi. Gọi $R$ là đối xứng của $P$ qua $A$ thì $OR=AQ$ không đổi, lại có $OR||AP\perp BC$ nên $AP$ luôn qua $R$ cố định $\blacksquare$

 

Thầy có lời giải nào mà để tránh tính toán phần sau không ạ ?

 

Hình vẽ bài toán:

Hình gửi kèm

  • Ảnh chụp Màn hình 2017-01-05 lúc 15.52.45.png



#667068 Đề Thi VMO năm 2017

Đã gửi bởi moonkey01 on 05-01-2017 - 13:13 trong Thi HSG Quốc gia và Quốc tế

Ý bạn là sao ? 

 

Do $B(x)$ có 2 nghiệm nên em không rõ là $Q(x)-(3x-1)$ có nhận số còn lại làm nghiệm hay không (em không rành lắm về đa thức). Nếu có, anh có thể chứng minh chặt chẽ luôn được không ạ ? 




#667064 Đề Thi VMO năm 2017

Đã gửi bởi moonkey01 on 05-01-2017 - 12:57 trong Thi HSG Quốc gia và Quốc tế

Câu 1:

Ý tưởng là để ý x = 3 là điểm nhạy cảm của dãy số.

a)Ta chứng minh các bước sau

- $x_n>3 \forall n$

- $|x_{n+1}-3|<\frac{|x_n-3|}{2}$

b)Chia làm hai TH:

Nếu tồn tại $n$ để $x_n>3$ thì cmtt như a)

Xét $x_n<3 \forall n$

Chọn dãy $v_n = \frac{1}{2} + \sqrt{2u_n+\frac{1}{4}}$, ta có $u_n>v_n$ và $lim v_n = 3$, suy ra $lim x_n = 3$ do bị kẹp

 

Câu 2:

Tồn tại

Chọn $A(x) = (x-1)^3-2$, $B(X) = x^2 + 2x - 4$, ta có:

$gcd(A,B) = 1$, $P(x) - x$ chia hết cho $A(x)$ và $Q(x) - (3x-1)$ chia hết cho $B(x)$

Suy ra, ta cần tìm $P(x)$ thoả mãn:

$P = A.Q + x = B.R + (3x-1)$

$\Leftrightarrow A.Q - B.R = 2x-1$

Vì $gcd(A,B) = 1$ nên theo thuật chia Euclid, tồn tại $Q,R$ hệ số nguyên thoả mãn, suy ra dpcm

 

$Q(x)-(3x-1)$ chia hết cho $B(x)$ thì mọi nghiệm của $B(x)$ là mọi nghiệm của $Q(x)-(3x-1)$, nhưng tại sao lại thế vậy anh ?




#666975 VMF's Marathon Hình học Olympic

Đã gửi bởi moonkey01 on 04-01-2017 - 21:05 trong Hình học

Lời giải sau do bạn Nguyễn Lê Phước gửi tới tác giả.

 

attachicon.gifFigure4051.png

 

Lời giải bài toán 101. Gọi $NI$ cắt $HK$ tại $S$, $MI$ cắt $HL$ tại $T$ thì $S,T$ là tâm bàng tiếp tam giác $ANH,AMH$. Từ đó $\angle ASE=45^\circ=\angle AHL$ và $\angle AES=90^\circ+\angle ANE=\angle ALH$. Từ đó hai tam giác $ASE$ và $ALH$ đồng dạng g.g. Lại có $\angle ATH=90^\circ-\frac{\angle AMN}{2}=180^\circ-\angle AIN=\angle AIS$. Từ đó hai tam giác $ATH$ và $AIS$ đồng dạng g.g. Vậy $\frac{ES}{EI}=\frac{LH}{LT}$. Tương tự $\frac{FT}{FI}=\frac{KH}{KS}$. Từ đó $\frac{FI}{FT}.\frac{LT}{LH}=\frac{EI}{ES}.\frac{KS}{KH}$. Dùng định lý Menelaus dễ thấy $FL,ES$ cùng đi qua một điểm trên $IH$.

 

Lời giải khác của bạn Nguyễn Đức Bảo có thể xem tại http://artofproblems...1308940p7009306, bài toán này nằm trong chuỗi bài mở rộng bài toán IMO 2009 của tác giả http://analgeomatica...-2009-ngay.html

 

Bài toán 102 (Tập huấn đội IMO 2016)Cho tam giác $ABC$ nội tiếp đường tròn $(O)$ và $P$ di chuyển trên cung $BC$ không chứa $A$. Đối xứng của $PA$ qua $PB$, $PC$ lần lượt cắt $AB$, $AC$ tại $F$, $E$. Một đường thẳng vuông góc với $PA$ tại một điểm chia $PA$ theo tỉ số cố định cắt tiếp tuyến tại $A$ của $(AEF)$ tại $Q$. Chứng minh rằng $Q$ luôn thuộc đường thẳng cố định khi $P$ thay đổi.

 

Trong quá trình cố gắng chứng minh bài toán, em phát hiện ra một số tính chất khác như sau. Không mất tính tổng quát, ta giả sử rằng $AB<AC$.

 

Tính chất 1: $EF$ đi qua $D$ là chân đường phân giác ngoài từ $A$ của $\Delta ABC$.

 

Thật vậy, do $NE$ là phân giác $\angle ANC$ nên $\frac{EC}{EA}=\frac{NC}{NA}=\frac{AC}{AH}$. Tương tự ta có $\frac{FA}{FB}=\frac{AH}{AB}$ nên nếu gọi $AD$ là phân giác ngoài của $\Delta ABC$ thì theo định lý Menelaus, chứng minh được $D,E,F$ thẳng hàng.

 

Tính chất 2: $NF,ME,AI$ đồng quy.

 

Từ tính chất 1 và lưu ý rẳng $AI$ là phân giác $\angle BAC$, lại có $AB,AC$ là hai đường đẳng giác trong $\Delta AMN$ nên gọi $AI$ cắt $BC$ tại $P$ thì $(DP,BC)=(DP,NM)=-1$. Mặt khác gọi $AI$ cắt $EF$ tại $S$ thì $A(DP,BC)=A(DS,FE)=(DS,FE)=-1=(DP,NM)$ nên $SP,NF,ME$ đồng quy hay $NF,ME$ cắt nhau trên $AI$.




#666626 VMF's Marathon Hình học Olympic

Đã gửi bởi moonkey01 on 02-01-2017 - 14:30 trong Hình học

Em đề nghị bài toán tiếp theo, đã được phát biểu lại cho gọn hơn. Bài toán trích từ đề thi chọn HSG lớp 9 trường THPT chuyên Trần Đại Nghĩa năm ngoái.

 

Bài toán 99: Cho tam giác $ABC$ có $O$ là tâm ngoại tiếp. Đường thẳng $d$ đi qua $O$ sao cho $B,C$ nằm cùng phía với $d$. Gọi $M,N$ là hình chiếu của $B,C$ trên $d$. Đường thẳng qua $M$ vuông góc $CA$ cắt đường thẳng qua $N$ vuông góc $AB$ tại $T$. Chứng minh rằng $T$ luôn thuộc đường tròn cố định khi $d$ thay đổi.

 

Theo cái nhìn chủ quan của em thì đây là một bài toán có cấu hình đẹp và thú vị, nên em rất mong thầy Hùng và các anh chị đóng góp những mở rộng của bài toán trên.




#666595 VMF's Marathon Hình học Olympic

Đã gửi bởi moonkey01 on 02-01-2017 - 12:18 trong Hình học

Bài toán 98. Cho tam giác $ABC$ giả sử có điểm $P$ nằm trong tam giác sao cho $\angle PBA=\angle PCA$ đồng thời nếu có các đường tròn $(K)$ qua $P,C$ và đường tròn $(L)$ qua $P,B$ sao cho $(K),(L)$ cắt nhau tại $Q$ khác $P$ thì $BQ$ đi qua giao điểm $E$ của $(K)$ và $AC$ còn $CQ$ đi qua giao điểm $F$ của $(L)$ và $AB$. Chứng minh rằng $BE=CF$.

 

attachicon.gifFigure4229.png

 

Nguồn gốc: http://www.cut-the-k.../Stoyanov.shtml

 

Lời giải của em như sau:

 

Không mất tính tổng quát, giả sử vị trí các điểm như hình vẽ thì ta có $\angle BFP = \angle BQP = \angle PCA = \angle PBA$ nên $\Delta PBF$ cân tại $P$ hay $PB=PF$. Tương tự ta có $PE=PC$ nên $\Delta PBE=\Delta PFC$ (c.g.c), dẫn đến $BE=CF$.




#662951 $\sum \frac{ab}{3+bc}\leq\frac...

Đã gửi bởi moonkey01 on 24-11-2016 - 21:29 trong Bất đẳng thức - Cực trị

Cho $a,b,c\geq 0$ thoả mãn $a+b+c=3$. Chứng minh rằng $\sum \frac{ab}{3+bc}\leq\frac{3}{4}$